50
$\begingroup$

Say $A$ and $B$ are symmetric, positive definite matrices. I've proved that

$$\det(A+B) \ge \det(A) + \det(B)$$

in the case that $A$ and $B$ are two dimensional. Is this true in general for $n$-dimensional matrices? Is the following even true?

$$\det(A+B) \ge \det(A)$$

This would also be enough. Thanks.

$\endgroup$
2
  • 1
    $\begingroup$ @RodrigodeAzavedo There is a geometric interpretation for this fact: If A and B are positive invertible matrices with $0<A<B$ then $Det A \leq Det B$. We interpret $A,B$ as inner products(Positive symmetric 2-tensores). Then there is a linear map $T$ with $T^*(B)=A)$ this means that T is metric decreasing. But every metric decreasing is a volum decreasing map. Since the n dimensional "Volume" is made by product of 1 dimensional length. $\endgroup$ Jun 30, 2018 at 10:37
  • $\begingroup$ @AliTaghavi Could you please explain what $T$ has got to do with the question? $\endgroup$ Dec 12, 2018 at 21:33

7 Answers 7

66
$\begingroup$

The inequality $$\det(A+B)\geq \det A +\det B$$ is implied by the Minkowski determinant theorem $$(\det(A+B))^{1/n}\geq (\det A)^{1/n}+(\det B)^{1/n}$$ which holds true for any non-negative $n\times n$ Hermitian matrices $A$ and $B$. The latter inequality is equivalent to the fact that the function $A\mapsto(\det A )^{1/n}$ is concave on the set of $n\times n$ non-negative Hermitian matrices (see e.g., A Survey of Matrix Theory and Matrix Inequalities by Marcus and Minc, Dover, 1992, P. 115 and also the previous MO thread).

$\endgroup$
9
  • 1
    $\begingroup$ looks nice, but isn't it somewhat overkill to invoke the Minkowski theorem here? $\endgroup$
    – Suvrit
    May 19, 2011 at 17:05
  • 51
    $\begingroup$ This is really misleading since it makes the question look like complicated, while is almost obvious that $\det(A+B) = \det(A) \det(1+A^{-1/2}BA^{-1/2}) \geq \det(A) (1+\det(A^{-1/2}BA^{-1/2})) = \det(A) + \det(B)$. In the second step, it is just used that $\prod_i (1+ \mu_i) \geq 1 + \prod_i \mu_i$, where $\mu_i$ are the eigenvalues of $A^{-1/2}BA^{-1/2}$. $\endgroup$ Jun 28, 2014 at 20:02
  • 10
    $\begingroup$ I'm going to disagree with @Suvrit and Andreas and say that the Minkowski theorem should be discussed. The reason is that it is a tighter bound that respects the dimensionality, and in particular is saturated when $A=B$ for any $n$. The OP's question is just an awkwardly weakened version of Minkowski. $\endgroup$ May 22, 2015 at 20:10
  • 3
    $\begingroup$ In fact, one can show a bit more with @AndreasThom idea: if $A$ is positive definite and $B$ is positive semidefinite and nonzero then $\det(A+B)>\det(A)+\det(B)$ (strict inequality), since $A^{-1/2}BA^{-1/2}$ is then nonzero positive semidefinite, hence with nonnegative eigenvalues and not all 0, and thus $\prod_i(1+\mu_i)\geq 1+\sum_i\mu_i+\prod_i\mu_i>1+\prod_i\mu_i$. $\endgroup$
    – Jose Brox
    Oct 10, 2017 at 18:44
  • 2
    $\begingroup$ I used $A(1+A^{-1/2}BA^{-1/2}) = A^{1/2}(A+B)A^{-1/2}$. For the second part, you would need to argue that the eigenvalues of $A^{-1}B$ are positive, which brings you back to my argument (I guess). $\endgroup$ Sep 26, 2019 at 6:03
51
$\begingroup$

We have $((A+B)x,x)\ge (Ax,x)$. It then follows from the variational characterization of eigenvalues (min-max theorem) that the eigenvalues of $A+B$ are greater than or equal to those of $A$. This implies $det(A+B)\ge det(A)$.

$\endgroup$
37
$\begingroup$

Here is yet another overkill, but hopefully not too bad a way to prove this inequality.

We have the following proof sketch.

$$\begin{eqnarray} x^T(A+B)x &\ge& x^TAx\quad\forall x\\\\ -x^T(A+B)x &\le& -x^TAx\\\\ \exp(-x^T(A+B)x) &\le& \exp(-x^TAx)\\\\ \int\exp(-x^T(A+B)x)dx &\le& \int\exp(-x^TAx)dx\\\\ \frac{1}{\sqrt{\det(A+B)}} &\le& \frac{1}{\sqrt{\det(A)}}\\\\ \det(A+B) &\ge& \det(A) \end{eqnarray} $$

The only fancy thing that happened is in the second last line, where I used the formula for the Gaussian integral (see multivariate section)


Update. To expand upon my comment below, to note that the above idea actually with a little bit more care actually yields a proof of the Minkowski determinant inequality, by equivalently establishing log-concavity of the determinant. The key point to observe is \begin{eqnarray} \exp(-x^T((1-\lambda)A+\lambda)x) &=& [\exp(-x^TAx)]^{1-\lambda}[\exp(-x^TBx)]^\lambda\\\\ \int\exp(-x^T((1-\lambda)A+\lambda)x)dx &=& \int [\exp(-x^TAx)]^{1-\lambda}[\exp(-x^TBx)]^\lambda\ dx\\\\ &\stackrel{\text{Hölder}}{\le}& \left(\int\exp(-x^TAx)dx \right)^{1-\lambda}\left(\int \exp(-x^TBx)dx \right)^\lambda. \end{eqnarray} Now invoke the Gaussian integral as above to conclude \begin{equation*} \det((1-\lambda)A+\lambda B) \ge \det(A)^{1-\lambda}\det(B)^\lambda, \end{equation*} from which we can easily conclude $\det(A+B)^{1/n} \ge \det(A)^{1/n}+\det(B)^{1/n}$.

$\endgroup$
2
  • $\begingroup$ This is very clever. I wonder if such reasoning can also prove the Minkowski determinant theorem mentioned above... $\endgroup$ Jun 30, 2018 at 22:35
  • 3
    $\begingroup$ @AlexArvanitakis indeed the same reasoning as above with a tiny bit of care can be used to prove the inequality $\det((1-\lambda)A+\lambda B) \ge \det(A)^{1-\lambda}\det(B)^\lambda$, which is actually equivalent to Minkowski's determinant inequality (easy to verify using suitably scaled versions of the matrices $A$ and $B$) $\endgroup$
    – Suvrit
    Jul 1, 2018 at 22:07
36
$\begingroup$

Yet another way to see this is to note that $A = \overline{Q}^{t}Q$ for some invertible matrix $Q$. Then ${\rm det}(A+B) = |{\rm det}(Q)|^{2}{\rm det}{( I + (\overline{Q}^{-1}})^{t}BQ^{-1})$.` Now $(\overline{Q}^{-1})^{t}BQ^{-1}$ is Hermitian, and positive definite. It suffices to prove that if $X$ is positive definite and Hermitian, then ${\rm det}(I+X) \geq (1 + {\rm det}X)$. We may conjugate $X$ by a unitary matrix $U$ and assume that $X$ is diagonal. Let the eigenvalues of $X$ be $\lambda_{1},\ldots, \lambda_{n}$, (allowing repetitions). Then ${\rm det}(I+X) = \prod_{i=1}^{n}(1 + \lambda_{i}) \geq 1 + \prod_{i=1}^{n} \lambda_{i} = 1 + {\rm det}X.$ Such an argument appears in some proofs by R. Brauer, though I do not know whether it originates with him.

Later edit: Incidentally, I think that with the arithmetic-geometric mean inequality and a slightly more careful analysis, you can see by this approach that for $X$ as above, you do have ${\rm det}(I+X) \geq (1 +({\rm det}X)^{1/n})^{n}$ (a special case of the inequality of Minkowski mentioned in the accepted answer, but enough to prove the general case by an argument similar to that above). For set $d = {\rm det}X$. Let $s_{m}(\lambda_{1},\ldots ,\lambda_{n})$ denote the $m$-th elementary symmetric function evaluated at the eigenvalues. Using the arithmetic-geometric mean inequality yields that $s_{m}(\lambda_{1},\ldots ,\lambda_{n}) \geq \left( \begin{array}{clcr} n\\m \end{array} \right)d^{m/n}$, so we obtain ${\rm det}(I+X) \geq (1+d^{1/n})^{n}.$

$\endgroup$
0
22
$\begingroup$

Let me add some more. If $A, B, C$ are positive semidefinite, then $$\det (A+B+C)+\det C\ge \det (A+C)+\det (B+C). \quad (\star)$$

When $C=0$, this reduces to OP's question.

A remarkable extension of ($\star$) were recently obtained by V. Paksoy, R. Turkmen, F. Zhang [ Electron. J. Linear Algebra 27 (2014) 332-341], which says that the determinant functional can be replaced by any generalized matrix function.

$\endgroup$
12
$\begingroup$

The determinant of a positive definite matrix $G$ is proportional to $(1/\hbox{Volume}(\mathcal B(G)))^2$ where $\mathcal B(G)$ denotes the unit ball with respect to the metric defined by $G$. If $A$ and $B$ are positive definite then the volume of $\mathcal B(A+B)$ is smaller than the volume of $\mathcal B(A)$ or $\mathcal B(B)$.

$\endgroup$
3
  • 2
    $\begingroup$ It's worth noting that this is secretly the same as Suvrit's answer. $\endgroup$ May 20, 2011 at 14:20
  • 4
    $\begingroup$ Not really: You don't need exponentials for proving that $\det(G)$ is proportional to $1/\hbox{Volume}(G)^2$ : It is enough to stare at an orthogonal basis formed of eigenvectors for $G$. In this sense this proof is more elementary. $\endgroup$ May 25, 2011 at 7:18
  • $\begingroup$ Fair enough.$ $ $\endgroup$ May 29, 2011 at 0:49
1
$\begingroup$

Just for those who might want to know, I think the min-max theorem mentioned by @Michael Renardy is Courant-Fischer: Supporse a real symmetric matrix A's eigen values are $\lambda_1 \leq \lambda_2 \leq ... \leq \lambda_n $. then k-th eigen value: $$ \lambda_k = \underset{V_k}{\text{min}} \space max\{x^TAx | x \in V_k, ||x||_2 = 1\}$$ $V_k$ is any k-dimentional subspace of $R^n$, $1 \leq k \leq n$ .

$\endgroup$

Your Answer

By clicking “Post Your Answer”, you agree to our terms of service and acknowledge you have read our privacy policy.

Not the answer you're looking for? Browse other questions tagged or ask your own question.